Plus And Minus

Calculus Level 2

1 + 1 1 + 1 1 + 1 1 + 1 + \dfrac1{1 + \dfrac1{1 + \dfrac1{1 + \ddots }}}

The value of the infintely nested fraction above is equal to 1 + 5 2 \dfrac{1+\sqrt5}2 . Find the value of the infinitely nested fraction below.

1 1 1 1 1 1 1 1 - \dfrac1{1 - \dfrac1{1 - \dfrac1{1 - \ddots }}}

Give your answer to 3 decimal places.

If you think that this number is a finite non-real number, submit your answer as 12345.

If you think that this number that does not converge (doesn't exist), submit your answer as 67890.


The answer is 67890.000.

This section requires Javascript.
You are seeing this because something didn't load right. We suggest you, (a) try refreshing the page, (b) enabling javascript if it is disabled on your browser and, finally, (c) loading the non-javascript version of this page . We're sorry about the hassle.

3 solutions

Pi Han Goh
May 5, 2016

Relevant wiki: Nested Functions

f ( k ) : = k 1 k 1 k 1 k f(k) : = \dfrac k{1 - \dfrac k {1 - \dfrac k{1 - \dfrac k {\ddots }}}}

We want to determine whether f ( 1 ) f(1) converge or not.

Suppose f ( k ) L f(k) \to L for some finite real L L , then L = k 1 L L = \dfrac k{1 - L } . Apply the quadratic formula gives L = 1 + 1 4 k 2 L = \dfrac{1 + \sqrt{1-4k}}2 . So f ( k ) f(k) only converges for k 1 4 k\leq \dfrac14 .

But for this question, k = 1 > 1 4 k = 1> \dfrac14 . So f ( 1 ) f(1) diverge.


This question was a direct reference from this discussion .

Aditya Oke
Apr 28, 2016

Square root of 3 + Square root of 3 + Square root of 3 ...... Like that how we solve same logic is applied here Assume that the denominator is x

1 + 1 ÷ 1 + 1÷ .....= x So equation is 1 + 1/x = x If we solve it we get X is 1+- √5/2.

doing the same for the other equation 1 - 1/x = x X^2 - x + 1 = 0 I got x is 1+- √-3/2 root of -3 is imaginary. Hence the answer is 67890. Real value does not exist.

Just list a few terms of the continued product. Some term will take the form 1 0 \dfrac 1 0 .

Hmmm, that's not a right interpretation. The nested fraction is infinitely nested! So there's no "last fraction".

Pi Han Goh - 5 years, 1 month ago

Log in to reply

Yes, but even if there was a last term, it would be of the form 1/0. So can't we conclude that it won't converge like this?

A Former Brilliant Member - 5 years, 1 month ago

Log in to reply

There is no last term. No, you can't conclude that way. One way to do it is to assume that it converges to L L , then find this value of L L . What do you get?

Pi Han Goh - 5 years, 1 month ago

Log in to reply

@Pi Han Goh If it converges to L, we get imaginary solutions. But since the nested fraction consists only of real numbers, it cannot be imaginary hence it does not converge.

Is it fine now sir?

A Former Brilliant Member - 5 years, 1 month ago

Log in to reply

@A Former Brilliant Member Yeah, it's fine now...

Pi Han Goh - 5 years, 1 month ago

0 pending reports

×

Problem Loading...

Note Loading...

Set Loading...